1 / 9

exercise of GTM052 1.1

solution of the exercise of GTM052 1.1

huguanggo
Télécharger la présentation

exercise of GTM052 1.1

An Image/Link below is provided (as is) to download presentation Download Policy: Content on the Website is provided to you AS IS for your information and personal use and may not be sold / licensed / shared on other websites without getting consent from its author. Content is provided to you AS IS for your information and personal use only. Download presentation by click this link. While downloading, if for some reason you are not able to download a presentation, the publisher may have deleted the file from their server. During download, if you can't get a presentation, the file might be deleted by the publisher.

E N D

Presentation Transcript


  1. 1.1 Guang Hu October 7, 2018 1. (a) Let Y be the plane curve y = x2(i.e. Y is the zero set of the polynomial f = y − x2). Show that A(Y ) is isomorphic to a polynomial ring in one variable over k. Proof. We define the homomorphism from A(Y ) = k[x1,x2] to k[x] by: f0(x) + f1(x)y + ... + fn(x)yn7→ f0(x) + x2f1(x) + ... + x2nfn(x) i.e. ϕ(f) is defined by ϕ(f)(x) = f(x,x2) This is obvious an epimorphism. We need to show that I(Y ) is the kernel of the map: If f(x,y) maps to the zero polynomial in k[x], then f(x,y) = 0 for any y = x2. So f ∈ I(Y ). If f ∈ I(Y ), then f(x,x2) for any x and thus the image of f is the zero polynomial (since k is infinite). (b) Let Z be the plane curve xy = 1. Show that A(Z) is not isomorphism to a polynomial ring in one variable over k. Proof. Suppose that such an isomorphism exists, which is denoted by ϕ. So we have ϕ(xy − 1) = 0 and hence ϕ(x)ϕ(y) = 1. So ϕ(x),ϕ(y) ∈ k. From the fact that x and y generates k[x,y] one can deduce the contradiction that im(ϕ) ⊂ k. (c) Let f be any irreducible quadratic polynomial in k[x,y], and let W be the conic defined by f. Show that A(W) is isomorphic to A(Y ) or A(Z). Which one is it when? 1

  2. Proof. We claim that for any non-degenerate (i.e. invertible) affine transfor- mation α, A(Y ) is isomorphic to A(αY ). Indeed, this property based on the fact that we can define fαin the following way: fα(x) := f(α−1x) such that fαis also a polynomial. Once we noticed this, we can define a map from A/I(Y ) to A/I(αY ) by: ¯f 7→¯fα Once can easily check that this is a well-defined isomorphism by definition. The rest things are simply linear algebra. 2. The Twisted Cubic Curve. Let Y ⊂ A3be the set Y = {(t,t2,t3)|t ∈ k}. Show that Y is a affine variety of dimension 1. Find generators for the ideal I(Y ). Show that A(Y ) is isomorphic to a polynomial ring in one variable over k. We say that Y is given by the parametric representation x = t,y = t2,z = t3. Proof. Y is an affine variety since Y = Z(f1)∩Z(f2) where f1= y −x2and f2= z − xy. Choose a nontrivial closed subset of Y , namely Y ∩Z(a) where a is generated by g1,g2,...,gn. Hence we have: Y ∩ Z(a) = Y ∩ (∪n i=1Z(gi)) = ∪n i=1(Y ∩ Z(gi)) For any (t,t2,t3) ∈ Y ∩ Z(gi), we have gi(t,t2,t3) = 0. So t is a solution of the one-variable polynomial gi(x,x2,x3) (which is not the zero polynomial since we cliamed that Y ∩Z(a) is non-trivial), which have only finitely many choice. So Y ∩ Z(gi) = 0 is a finite set and thus Y ∩ Z(a) is a finite set. Now we have proved that any non-trivial closed subset of Y is nothing but a set of finite points. So the only non-trivial irreducible closed sets are sets of a single point, so dim(Y ) = 1. We are going to prove that k[x,y,z]/(y −x2,z −x3) is isomorphic to k[x]. If this is right, then (y − x2,z − x3) is prime and hence equal to I(Y ). Let’s construct a homomorphism of rings: ϕ : k[x,y,z] −→ k[x] f(x,y,z) 7−→ f(x,x2,x3) 2

  3. It is obvious that ϕ is surjective and (y − x2,z − x3) ⊂ kerϕ To show that kerϕ ⊂ (y−x2,z −x3), we only need to expansion f(x,y,z) = f(x,y −x2+x2,z −x3+x3) by regarding y −x2z −x3as variables, leading to the follwing equation: f(x,y,z) =¯f + i(x,y,z) where¯f ∈ k[x]. If 0 = ϕ(f) = ϕ(¯f) + ϕ(i) = ϕ(¯f), we have¯f = ϕ(¯f) = 0, hence f ∈ (y − x2,z − x3). 1.3 Let Y be the algebraic set in A3defined by the two polynomials x2−yz and xz−x. Show that Y is a union of three irreducible components. Describe them and find their prime ideals. Proof. One can easily check that Y = Z(a1)∪Z(a2)∪Z(a3), where a1= (x,y), a2= (x,z), a3= (y − x2,z − 1). It’s easy to show that k[x,y,z]/ai∼= k[x]. Hence aiis prime and Z(ai) is irreducible. 1.4 If we identify A3with A1×A1in the natural way, show that the Zariski topology on A2is not the product topology of the Zariski topologies on the two copies of A1. Proof. The non-trivial closed set in the product topology of A1×A1is noth- ing but finite union of {(x,y) : x = c} or {(x,y) : y = c}. So we can see {(x,y) : x = y} is closed in the Zariski topology of A2, while not closed in the product topology. 1.5 Show that a k−algebra B is isomorphic to the affine coordinate ring of some algebraic set in An, for some n, if and only if B is finitely generated k−algebra with no nilpotent elements. Proof. We have known it in Remark 1.4.6 that any k− algebra B is isomor- phic to the affine coordinate ring of some algebraic set if and only if B is finitely generated and is a domian. Further more, write B as the quotient of a polynomial ring k[x − 1,...,xn]/a. If B have no nilpotent elements, then fm∈ a implies that f ∈ a. So√a = a and hence A/I(Z(a)∼= A/a = B. 3

  4. 1.6 Any nonempty open subset of an irreducible topological space is dense and irreducible. If Y is a subset of a topological space X, which is irreducible in its induced topology, then the closure¯Y is also irriducible. Proof. For any two open subsets U1,U2of an irreducible topological space X, Uc Suppose that U1 ⊂ V1∪ V2 where V1 and V2 are both closed. We have (Uc to say, U1⊂ V1or V2= X ⇒ U1⊂ V2. So U1is also irreducible. Now we come to the second statement. Suppose that¯Y ⊂ V1∪ V2where V1,V2is closed in X. So Y ⊂ V1∪ V2and hence Y ⊂ V1or Y ⊂ V2. So¯Y ⊂ V1of¯Y ⊂ V2by the universal property of closure. 2are not the whole space, thus Uc 2̸= X. So U1∩ U2̸= ∅. 1,Uc 1∪ Uc 1∪V1= X or V2= X. That is 1∪V1)∪V2= X. Since X is irreducible, Uc 1.7 (a) Show that the following conditions are equivalent for a topological space X: (i) X is noetherian; (ii) every nonempty family of closed subsets has a minimal element; (iii) X satisfies the ascending chain condition for open subsets; (iv) every nonempty family of open subsets has a minimal element. Proof. (i)⇒(ii) If there is exist a nonempty family S of closed subsets without a minimal element, i.e. for any subset in the family S, there is a smaller element in S. So we can find a descending chain which is not eventually stationary by induction. (ii)⇒(i) The family of subsets in the descending chain has a minimal element, so the chain is stationary after that element. It is alternative that (i) is equivalent to (iii) and (ii) is equivalent to (iv) (b) A noetherian topological space is quasi-compact, i.e. every open cover has a finite subcover. Proof. Suppose that ∪i∈IUiis an open cover of the topological space. Let S be the collection of all the finite union for some Ui. From a maximal element in S, we get a finite subcover we want. (c) Any subset of a noetherian topological space is noetherian in its induced topology. 4

  5. Proof. Let X be a noetherian topological space and Y its subspace. A de- scending chain of closed set in Y is Y1∩ Y ⊃ Y2∩ Y ⊃ ... Since we have a descending chain of closed set: Y1⊃ Y1∩ Y2⊃ ... There is a minimal element in it, denoted by ∩n i=1Yi. So for any m > n we have ∩n Yn∩ Y = ∩n i=1Yi⊂ Ym. Hence i=1(Yi∩ Y ) = (∩n i=1Yi) ∩ Y ⊂ Ym∩ Y (d) A noetherian space which is also Hausdorff must be a finite set with the discrete topology. Proof. Form proposition 1.5 we know that a noetherian space X itself can be expressed as a finite union X = Y1∪Y2...∪Ynof irreducible closed subsets.For any i = 1,2,...,n, if Yicontains more than 1 elements, then they can sepa- rated by two open sets in X, namely U1,U2. Hence (U1∩Yi)∩(U2∩Yi) ̸= ∅ while neither of them is empty, that’s a contradiction. So every Yihas only 1 element and thus X is finite. So the topology in X is discrete since X is Hausdorff. 1.8. Let Y be an affine variety of dimension r in An. Let H be a hypersurface in An, and assume that Y ⊈ H. Then every irreducible component of Y ∩H has dimension r − 1 Proof. Suppose that H is the hypersurface defined by f = 0. And we may also assume that Y ∩H ̸= ∅, hence f +I(Y ) is not a unit in A(Y ) = A/I(Y ). Since Y ⊈ H, we have f / ∈ I(Y ) and hence f +I(Y ) is not the zero in A(Y ). Since Y is irreducible, I(Y ) is prime and hence f is not a zero divisor in A(Y ). Then let’s consider an irreducible components U of Y ∩ H. Define J(U) := {g + I(Y ) ∈ A(Y ) : g(U) = 0}. 5

  6. J(U) is well-defined since g ∈ I(Y ) ⇒ g(U) ⊂ g(Y ) = 0 We claim that J(U) is a minimal prime ideal containing f + I(Y ) in A(Y ). In fact, from f(U) ⊂ f(H) = 0 we know that f + I(Y ) ∈ J(U). If we have (g1+ I(Y ))(g2+ I(Y )) := g1g2+ I(Y ) ∈ J(U) while neither of them is in J(U). That is to say, g1g2∈ I(U). Since U is irreducible, I(U) is prime. So g1∈ I(U) or g2∈ I(U), namely g1+ I(Y ) ∈ J(U) or g2+ I(Y ) ∈ J(U). So J(U) is prime. To see that J(U) is minimal of f + I(Y ), suppose that there is a prime ideal p such that f + I(Y ) ∈ p ⊊ J(U), and define the retraction of ideals: pc:= {g ∈ A : g + I(Y ) ∈ p}. It obvious that f ∈ pc⊊ I(U). Recall that the retraction of a prime ideal is also prime, we can deduce that U = Z(I(U)) ⊊ Z(pc) ⊂ Z(f) = H It’s trivil that I(Y ) ⊂ pcsince 0 ∈ p. So Z(pc) ⊂ Z(I(Y )) = Y . Hence Z(pc is an irreducible subset of Y ∩ H strictly containing U, this contradicts to the assumption that U is an irreducible component. Now use Krull’s Hauptidealsatz, J(U) has height 1. From theorem 1.8A (b) we can deduce that dimA(Y )/J(U) = dimA(Y ) − 1 = r − 1. The final step is to prove that A(Y )/J(U)∼= A/I(U). After constructing the following homomorphism of rings: A −→ A(Y )/J(U) f 7−→ (f + I(Y )) + J(U) we can easily find that the kernel of the homomorphism is I(U), since f + I(Y ) ∈ J(U) means that f(U) = 0, namely f ∈ I(U), by definition. 1.9. Let a be an ideal which can be generated by r elements. Then every irreducible component of Z(a) has dimension ≥ n − r. Proof. Use the induction on r, if a can be generated by r + 1 elements, namely f1,f2,...fr,fr+1. Since Z(a) = Z(f1,...,fr)∩Z(fr+1), any irreducible component Y in Z(a) is in Z(f1,...,fr), thus in some irreducible component 6

  7. of Z(f1,...,fr), namely Y′. So Y ⊂ Y′∩Z(fr+1), and in fact Y is a irreducible component of Y′∩ Z(fr+1). Then use 1.8, and the induction assumption. If Y′⊂ Z(fr+1), Y′= Y and hence dimY′= dimY ≥ n − r. If Y′⊈ Z(fr+1), then dimY = dimY′− 1 ≥ n − (r + 1) When r = 0, Z(a) =An, so dimZ(a) = n ≥ n − r. 1.10. (a) If Y is any subset of a topological space X, then dimY ≤ dimX. Proof. If Y0 ⊂ Y1 ⊂ ... ⊂ Yn is a sequence of distinct closed irreducible subsets of Y , then¯Y0 ⊂¯Y1 ⊂ ... ⊂¯Yn is a sequence of closed irreducible subsets of X by 1.6. For any i = 0,1,...,n, Yi= V ∩ Y for some V closed in X. V ∩ Y ⊂¯Yiand V ∩Y ⊂ Y together implies that V ∩Y ⊂¯Yi∩Y . At the same time, Yi⊂ V implies that¯Yi⊂ V , thus¯Yi∩ Y ⊂ V ∩ Y . So¯Yi∩ Y = V ∩ Y = Yi. If¯Yi =¯Yj for some i ̸= j, then Yi =¯Yi =¯Yj = Yj, contradiction. So ¯Y0⊂¯Y1⊂ ... ⊂¯Ynis a distinct sequence. (b) If X is a topological space which is covered by a family of open subsets{Ui}, then dimX = supdimUi. Proof. dimX ≥ dimUifor all i by (a), so dimX ≥ supdimUi. If X0⊂ X1⊂ ... ⊂ Xnis a sequence of distinct closed irreducible subsets of X, we choose a U (equal to some Ui) such that X0∩ U ̸= ∅. So X0∩ U ⊂ X1∩U ⊂ ... ⊂ Xn∩U is a sequence of distinct closed subsets of U. From 1.6 we know that Xi∩U is irreducible and dense in Xi. Since Xc open subset in Xi, (Xi∩U)∩(Xc not contained in Xi−1and hence the sequence X0∩U ⊂ X1∩U ⊂ ... ⊂ Xn∩U is distinct. i−1∩Xi̸= ∅ is an i−1̸= ∅. So Xi∩U is i−1∩Xi) = (Xi∩U)∩Xc (c) Give an example of a topological space X and a dense open subset U with dimU < dimX. Let X = 0,1 the topology of X is defined by given the open sets: ∅,X,U = {0}. Then U is dense in X while dimX = 2,dimU = 1. (d)If Y is a closed subset of an irreducible finite-dimensional topologi- cal space X, and if dimY = dimX then Y = X. 7

  8. Proof. If Y ̸= X, and dimY = n, then for any sequence of distinct closed irreducible subsets: Y0 ⊂ Y1 ⊂ ... ⊂ Yn, Y0 ⊂ Y1 ⊂ ... ⊂ Yn ⊂ X is a sequence of disdinct closed irreducible subsets of X, so dimX > dimY , a contradiction. (e) Give an example of a noetherian topological space of infinite dimen- sion. The cofinite topology in any infinite set. 1.11. Let Y ⊂ A3be the curve given parametrically by x = t3,y = t4,z = t5. Show that I(Y ) is a prime ideal of height 2 in k[x,y,z] which cannot be gen- erated by 2 elements. We say Y is not a local complete intersection–cf.(Ex. 2.17). Proof. Considerating the following ring homomorphism from k[x,y,z] to k[t]. f(x,y,z) 7−→ f(t3,t4,t5) The kernel of this homomorphism is precisely I(Y ), so A(Y ) is isomorphic to a subring of k[x], hence the transcendence degree of its quotient field (as a intermediate filed of k and k(x) is 0 or 1. However, t3is not in k, so the transcendence degree is 1, thus the height of I(Y ) is 2. Suppose that I(Y ) is generated by 2 elements, f and g,we need to deduce a contradiction. First we need to prove that for any polynomial h ∈ I(Y ), the coefficient of x,x2,y,z are all equal to zero. Note that is obvious that h has no constant term. i) Denote h by h = ax+x2h1+yh2+zh3, then from h(t3,t4,t5) = 0 we know that at3+ t6h1+ t4h2+ t5h3= 0, the only terms of t3appears at the first so a = 0. ii) For h = ax2+ x3h1+ yh2+ zh3, we have at6+ t9h1+ t4h2+ t5h3= 0, and hence a = 0. Note that we use the same symble because we use them as disposable. iii) h = ay+y2h1+xh2+zh3, then at4+t8h1+t3h2+t5h3= 0, hence a = 0. iv) h = az+z2h1+xh2+yh3, then at5+t10h1+t3h2+t4h3= 0, hence a = 0. Then, since it is obvious that x3− yz,y2− xz,z2− x2y are all in I(Y ), we have  u v    [f ] x3− yz y2− xz z2− x2y p q r  ·   s = g 8

  9. where p,q,r,s,u,v ∈ k[x]. Suppose that the coefficients of terms x3,y2,z2in f and g are respectively fx,fy,fzand gx,gy,gz, and the constant term of p,q,r,s,u,v are respectively p0,q0,r0,s0,u0,v0. It easy to verify that  u0 v0    [fx fy ] p0 r0 q0 s0 1 0 0 0 1 0 0 0 1 fz gz  ·   = gx gy which is a contradiction. 1.12. Give an example of an irreducible polynomial f ∈ R[x,y], whose zero set Z(f) in A2 Ris not irreducible (cf. 1.4.2). f = (x2− 1)2+ y2. 9

More Related